math

New Member
ارسال ها
1,129
لایک ها
1,096
امتیاز
0
پاسخ : ماراتن نظریه ی اعداد (سطح ممتاز)

فیبوناتچی+استقرا


لینک ندید.
این سوال به اتسقرا مربوط نمیشه کاقیه از یه اتحاد فیبوناچی استفاده کنید
 

math

New Member
ارسال ها
1,129
لایک ها
1,096
امتیاز
0
پاسخ : ماراتن نظریه ی اعداد (سطح ممتاز)

ایده ی این سوالا عموما اینطوریه که حدس می زنیم. الانم من فقط با دیدن جملات و حدس زدن اینا رو بدست آوردم:

تعریف کنید
و ثابت کنید
بله به طور ساده تر برای هر n داریم
 
ارسال ها
337
لایک ها
82
امتیاز
0
پاسخ : ماراتن نظریه ی اعداد (سطح ممتاز)

2. aوbوc صحیحند و
صحیح شده. ثابت کنید abc مکعب کامله.





بسیار خلاصه گویی کردم اما خب سوال باحالی بود سوال یکم به دلیل نوشتن داشتن زیاد یکی دیگه بذاره چون اون قبلا حل کردم حال نوشتنش ندارم
 

math

New Member
ارسال ها
1,129
لایک ها
1,096
امتیاز
0
پاسخ : ماراتن نظریه ی اعداد (سطح ممتاز)

ایده ی این سوالا عموما اینطوریه که حدس می زنیم. الانم من فقط با دیدن جملات و حدس زدن اینا رو بدست آوردم:

تعریف کنید
و ثابت کنید



گرچه خلاف قوانین ماراتنه ولی چون من کم ایجا سر میزنم و بچه ها خیلی از سوت و کور بون اینجا مینالند بیش از یه سوال میذارم. ببخشید!

ایده ی سوالا تقریبا شبیه همه . امیدوارم به دردتون بخوره

1. برای هر n طبیعی ثابت کنید :


2. aوbوc صحیحند و
صحیح شده. ثابت کنید abc مکعب کامله.


3. a عددی طبیعیه و n طبیعی و بزرگتر از یکه و برای هر k طبیعی عدد طبیعی
موجوده که
. ثابت کنید a توان nام کامله.

سوال اول :

از این نکته استفاده کنید که


و تعداد P را در دو طرف بشمارید

سوال سوم :

a رو تجزیه کنید
حال میدانیم
به راحتی ثابت میشود که توان های عوامل a بر n بخش پذیر اند
 
ارسال ها
337
لایک ها
82
امتیاز
0
پاسخ : ماراتن نظریه ی اعداد (سطح ممتاز)

یه سوال که ایده ی حلش با حاله :

x[SUP]2[/SUP](y-1)+y[SUP]2[/SUP](x-1)=1

این معادله رو در اعداد صحیح حل کنید سوالش راحته فقط یه ایده داره که قشنگه به خاطر همین دادمش
 

math

New Member
ارسال ها
1,129
لایک ها
1,096
امتیاز
0
پاسخ : ماراتن نظریه ی اعداد (سطح ممتاز)

یه سوال که ایده ی حلش با حاله :

x[SUP]2[/SUP](y-1)+y[SUP]2[/SUP](x-1)=1

این معادله رو در اعداد صحیح حل کنید سوالش راحته فقط یه ایده داره که قشنگه به خاطر همین دادمش
از این نکته استفاده کنید که :

اگر



اگر معادله رو ساده کنیم داریم
 

ash1374

New Member
ارسال ها
253
لایک ها
422
امتیاز
0
پاسخ : ماراتن نظریه ی اعداد (سطح ممتاز)

ایده ی هر سه سوال قبل شمردن عوامل p بود.

سه تا سوال دیگه: ( اینا ایده هاشون ربطی به هم نداره. فقط در خرکاری داشتن مشترکند)

1. (بامزه) 10 عدد طبیعی دوبه دو متمایز بیابید که هم جمعشان و هم ضربشان مکعب کامل شوند.

2. ( بی مزه ولی آموزنده) همه ی k های طبیعی رو بیابید که معادله ی
در اعداد طبیعی جواب داشته باشه.

3. اگر aوb طبیعی باشند و
مکعب کامل باشه ثابت کنید a=b.

اگر این سوالا تا شب حل شدن سه سوال دیگه میذارم انشاالله. بعدش دیگه خودتون ادامه بدید و نذارید اینجا بخوابه.
 
ارسال ها
317
لایک ها
151
امتیاز
0
پاسخ : ماراتن نظریه ی اعداد (سطح ممتاز)

ایده ی این سوالا عموما اینطوریه که حدس می زنیم. الانم من فقط با دیدن جملات و حدس زدن اینا رو بدست آوردم:

تعریف کنید
و ثابت کنید



گرچه خلاف قوانین ماراتنه ولی چون من کم ایجا سر میزنم و بچه ها خیلی از سوت و کور بون اینجا مینالند بیش از یه سوال میذارم. ببخشید!

ایده ی سوالا تقریبا شبیه همه . امیدوارم به دردتون بخوره

1. برای هر n طبیعی ثابت کنید :


2. aوbوc صحیحند و
صحیح شده. ثابت کنید abc مکعب کامله.


3. a عددی طبیعیه و n طبیعی و بزرگتر از یکه و برای هر k طبیعی عدد طبیعی
موجوده که
. ثابت کنید a توان nام کامله.
با عرض ادب خدمت استاد شاولی

حدس برای حل این دنباله ها وقت گیره این سوالها دوراه خیلی سر راس داره که بدون هیچ خلاقییت خاصی حل میشه :93:
1- استفاده از معادله پل
2-استفاده از چند جمله ایه چبشف

منبع __ کتاب polynomials مولف victor parasolov بخش چندجمله ای چبشف
 

zz_torna2

New Member
ارسال ها
300
لایک ها
254
امتیاز
0
پاسخ : ماراتن نظریه ی اعداد (سطح ممتاز)

این سوال به اتسقرا مربوط نمیشه کاقیه از یه اتحاد فیبوناچی استفاده کنید
منظورتون این اتحاده استاد؟


:3:
 
لایک ها MBGO

math

New Member
ارسال ها
1,129
لایک ها
1,096
امتیاز
0

zz_torna2

New Member
ارسال ها
300
لایک ها
254
امتیاز
0
پاسخ : ماراتن نظریه ی اعداد (سطح ممتاز)

ایده ی هر سه سوال قبل شمردن عوامل p بود.

سه تا سوال دیگه: ( اینا ایده هاشون ربطی به هم نداره. فقط در خرکاری داشتن مشترکند)

1. (بامزه) 10 عدد طبیعی دوبه دو متمایز بیابید که هم جمعشان و هم ضربشان مکعب کامل شوند.

2. ( بی مزه ولی آموزنده) همه ی k های طبیعی رو بیابید که معادله ی
در اعداد طبیعی جواب داشته باشه.

3. اگر aوb طبیعی باشند و
مکعب کامل باشه ثابت کنید a=b.

اگر این سوالا تا شب حل شدن سه سوال دیگه میذارم انشاالله. بعدش دیگه خودتون ادامه بدید و نذارید اینجا بخوابه.
س2:
به ازای تمام k به جز 2و3 معدله جواب داره.که بر اساس زوجیت k یکی ازجواب ها اینا هستند:
 

ash1374

New Member
ارسال ها
253
لایک ها
422
امتیاز
0
پاسخ : ماراتن نظریه ی اعداد (سطح ممتاز)

راهنمایی سوال یک: از رابطه ی معروف
می تونین استفاده کنید.


س2:
به ازای تمام k به جز 2و3 معدله جواب داره.که بر اساس زوجیت k یکی ازجواب ها اینا هستند:
ای کاش روش رسیدن به این جواب ها رو مینوشتین. به هر حال ایده ی من اینه که اساسا تو این سوال خیلی دستمون بازه. xوy دست ماست. حتی میتونیم متغیر اضافه کنیم. معادله رو به فرم
بنویسید تا k آزاد تر باشه. بعد قرار بدید y=tm و...

2 تا ار راه حل ها می تونه اینا باشه :

استفاده از

یا
پودر کردن مسئله با LFT !!!
با تشکر از استاد. فکر نمی کردم به این سادگی ها به قاف( قضیه آخر فرما) تبدیل بشه. تو ذهنم اینطوری بود که باید یه مقدار خوبی بنویسیم تا به قاف برسیم. برای همین گفتم خرکاری داره. راه
هم که عملا همون روش اثبات قاف برای n=3 هست.
 

ash1374

New Member
ارسال ها
253
لایک ها
422
امتیاز
0
پاسخ : ماراتن نظریه ی اعداد (سطح ممتاز)

دو تا سوال ساده تر:

1. r عددی گویا و مثبت است.
هم گویاست. ثابت کنید r طبیعیه.

2. در دنباله ی a[SUB]n[/SUB] از اعداد طبیعی
. ثابت کنید
 
ارسال ها
337
لایک ها
82
امتیاز
0
پاسخ : ماراتن نظریه ی اعداد (سطح ممتاز)

ببخشید دوستان یه سوال داشتم گفتم موضوع جدایی ایجاد نکنم با توجه به این که دو تا سوالم استاد ash1374 دادن میتونید رو این سه تا فکر کنید

تمام اعداد سه رقمی را بیابید که بر یازده بخش پذیر باشند و n/11 که n عدد سه رقمی ماست برابر مربغ ارقام n باشد من که یه راه حل دادم در اومد فقط 550 نمیدونم بازم هست یا نه برای همین گفتم شما یه کمکی کنید
 

ash1374

New Member
ارسال ها
253
لایک ها
422
امتیاز
0
پاسخ : ماراتن نظریه ی اعداد (سطح ممتاز)

الآن مشکل چیه که کسی بعد چند روز جواب نگذاشته.

-سوالا خیلی بدیهیند
-سوالا خیلی سختند
-سوالا خیلی مزخرفند
-کسی حوصله نداره سوالا رو حل کنه
-اصلا کسی وجود نداره که سوالا رو حل کنه
و...

به هر حال من طبق سنت راهنمایی میذارم

1.راهنمایی: اگر عددی طبیعی هم توان nام باشد هم mام و
آنگاه توان nmام کامل هم هست.

2. راهنمایی: ثابت کنید
و بعد برهان خلف بزنید.( برهان خلف روی همانی نبودن)
 

math

New Member
ارسال ها
1,129
لایک ها
1,096
امتیاز
0
پاسخ : ماراتن نظریه ی اعداد (سطح ممتاز)

من یه سوال تقریبا کلاسیک :188: برای راه افتادن تاپیک میزارم امید وارم راه بیافته


ثابت کنید حاصل ضرب هیچ دو دستگاه کامل مانده ها به هنگ m تشکیل دستگاه کامل مانده ها به پیمانه m نمیدهد
 

Armin_sf

New Member
ارسال ها
84
لایک ها
21
امتیاز
0
پاسخ : ماراتن نظریه ی اعداد (سطح ممتاز)

اين سوال تو ميرزاخاني هست ولي من جوابشو نفهميدم يكي توضيح بده لطفا
 

sima98

New Member
ارسال ها
95
لایک ها
39
امتیاز
0
پاسخ : ماراتن نظریه ی اعداد (سطح ممتاز)

من یه سوال تقریبا کلاسیک :188: برای راه افتادن تاپیک میزارم امید وارم راه بیافته


ثابت کنید حاصل ضرب هیچ دو دستگاه کامل مانده ها به هنگ m تشکیل دستگاه کامل مانده ها به پیمانه m نمیدهد
با استفاده از قضیه ویلسون
و چند لم ساده ی همنهشتی اثبات میشه!
 

mehrdad1st

New Member
ارسال ها
93
لایک ها
40
امتیاز
0
پاسخ : ماراتن نظریه ی اعداد (سطح ممتاز)

میشه راه حلتون رو بذارید چون من فقط توی یه حالت خاص تونستم اثباتش کنم:150:
 
بالا